-
Notifications
You must be signed in to change notification settings - Fork 1
/
Copy pathHomework05-1.tex
75 lines (63 loc) · 2.62 KB
/
Homework05-1.tex
1
2
3
4
5
6
7
8
9
10
11
12
13
14
15
16
17
18
19
20
21
22
23
24
25
26
27
28
29
30
31
32
33
34
35
36
37
38
39
40
41
42
43
44
45
46
47
48
49
50
51
52
53
54
55
56
57
58
59
60
61
62
63
64
65
66
67
68
69
70
71
72
73
74
75
\section{0316}
\begin{questions}
\question 求解递推关系$$T(n) = n + \sum_{i=1}^{n-1}{T(i)}$$其中$T(1)=1$
\begin{solution}
$$
T(n+1) = (n+1) + \sum_{i=1}^{n}{T(i)}
$$
$$
T(n+1) - T(n) = \left[(n+1) + \sum_{i=1}^{n}{T(i)}\right] - \left[n + \sum_{i=1}^{n-1}{T(i)}\right]
= 1 + T(n)
$$
$$
T(n+1) = 2T(n) + 1 \Rightarrow
T(n+1) + 1 = 2T(n) + 2 \Rightarrow
\frac{T(n+1) + 1}{T(n) + 1} = 2
$$
所以当$n>2$时,
$$
T(n) + 1 = T(1) \times 2^n \Rightarrow
T(n) = 2^n - 1
$$
又当$n=1$时,$T(1) = 1$恰好符合上式,所以对任意的$n \in \mathbb{N}^*$,
$$
T(n) = 2^n - 1
$$
\end{solution}
\question 在寻找一对一映射问题中,算法结束时集合S是否可能为空集?请证明你的结论。
{\kaishu
给定一个集合$A$和一个从$A$到自身的映射$f$,
寻找一个元素个数最多的子集$S \subseteq A$,满足$f$在$S$上是一一映射
}
\begin{solution}
不可能为空集。
\begin{proof}
设集合$A$中有$n$个元素。
从集合中任取一个元素记作$x_1$,记$i \geq 2$时$x_{i} = f(x_{i-1})$。
由于对任意的$x \in A$,都有且仅有一个$x' \in A, x' = f(x)$,所以序列$\left\{x_i\right\}$的长度是无限的。
但是集合$A$中元素个数是有限的,所以必然存在至少一对$m < n$使得$x_m = x_n$。
取一对使$x_m, \dots, x_{n-1}$各不相同的$m,n$
\begin{itemize}
\item 若存在$k:m < k < n , x_m = x_k$,则令$n = k$
\item 若存在$p,q:m < p < q < n , x_p = x_q$,则令$m = p, n = q$
\end{itemize}
由$\left\{x_i\right\}$的定义可知,$x_m, \dots, x_{n-1}$构成了一个循环置换
$$
\left(
\begin{array}{ccc}
x_m & \cdots & x_{n-1}
\end{array}
\right)
=
\left(
\begin{array}{cccc}
x_m & x_{m+1} & \cdots & x_{n-2} \\
x_{m+1} & \cdots & \cdots & x_{n-1}
\end{array}
\right)
$$
由置换的定义可知,$f$在$\left\{x_m, \dots , x_{n-1} \right\}$上是一一映射,
即$\left\{x_m, \dots , x_{n-1} \right\} \subseteq S$,所以$S \neq \Phi$。
\end{proof}
\end{solution}
\end{questions}